Calc 3 Easy Questions: Partial Derivative of g(x,y)

  • Thread starter Thread starter don23
  • Start date Start date
  • Tags Tags
    Calc 3
Click For Summary
To find the partial derivative of the function g(x,y) = x^2 * e^(-y) with respect to y, the limit definition can be applied. The expression simplifies to lim as h approaches 0 of [(x^2 * e^(-(y+h))) - (x^2 * e^-y)]/h. Factoring out x^2 * e^(-y) allows for the limit to focus on lim(e^(-h) - 1)/h, which is indeterminate. Applying l'Hospital's rule helps resolve this limit, resulting in a final value of -x^2 * e^(-y). Understanding this process can clarify the application of both the limit definition and l'Hospital's rule in calculating partial derivatives.
don23
Messages
9
Reaction score
0
calc 3 easy question!

I am trying to find the partial derivative of the following function with respect to y. I know how to find it without using the definition...but i want to know how to do it both ways. any help??

g(x,y)=x^2*e^-y

I got: lim as h approaches h [(x^2*e^-(y+h))-x^2*e^-y]/h...simplifying it is the hard part. ...any step by step help would be greatly appreciated.
 
Physics news on Phys.org
I think you need to use l'Hospital's rule for limits (heard of it?). Anyways, from the point you got to, you can factor x^2*e^(-y) from the expression, and since the only variable with respect to the limit is h, you can take x^2*e^(-y) out of the limit.

Then you're left with lim(e^(-h)-1)/h, which is indeterminate (0/0). Here's where you can use l'Hospital's rule, which states that for an indeterminate ratio of two expressions, the limit of the ratio is equal to the limit of the ratio of the derivatives. Once you do this, the limit is quite friendly, and you can see it is actually equal to -1.

Helpful?
 
Last edited:
Question: A clock's minute hand has length 4 and its hour hand has length 3. What is the distance between the tips at the moment when it is increasing most rapidly?(Putnam Exam Question) Answer: Making assumption that both the hands moves at constant angular velocities, the answer is ## \sqrt{7} .## But don't you think this assumption is somewhat doubtful and wrong?

Similar threads

  • · Replies 4 ·
Replies
4
Views
1K
  • · Replies 8 ·
Replies
8
Views
2K
  • · Replies 6 ·
Replies
6
Views
2K
Replies
4
Views
2K
  • · Replies 3 ·
Replies
3
Views
1K
  • · Replies 4 ·
Replies
4
Views
1K
Replies
2
Views
2K
  • · Replies 18 ·
Replies
18
Views
3K
  • · Replies 2 ·
Replies
2
Views
1K
Replies
9
Views
2K